September 16, 2009

Wednesday's questions and sample explanations - #8

8. The proposal to reduce grain prices by having the government set and control the price of grain would __________. Artificially stabilized pricing drains the economy, encourages inefficient production, and discourages individual initiative. If we let the forces of the free market operate, we give the more efficient producers the advantage they deserve.

Which one of the following most logically fills the blank of the passage above?

  1. reward producers who are not responding adequately to market forces
  2. actually have the opposite effect of increasing grain prices
  3. strike a necessary balance between government intervention and a free market economy
  4. make grain available to those who cannot afford to eat well
  5. allow individual efforts to influence the operation of the free market
(A) is correct. The question asks us to determine what the author's argument is. Therefore it is an inference question. The opening sentence, with the blank at the end, presents us with a proposal to accomplish a certain goal (lowering grain prices) by doing a particular thing (having the government set the price). The presence of the blank after the word "would" implies that the blank will contain the author's suggestion of what the result would be if we try to solve that problem by doing that thing. The rest of the stimulus suggests that he is against it because he believes in free-market principles, specifically the idea that those who succeed do so because they are efficient at producing their products. (A) is therefore the best answer because it reflects the free-market conservative concern that government intervention will help those who don't deserve to succeed.

(B) is incorrect even though we often hear claims that proposals for government action will have the exact opposite effect of what is intended. The author in this case is less concerned with grain prices than he is with free-market philosophy. He doesn't care if prices go up; he wants to make sure that only those who deserve an advantage, have an advantage.

(C) is incorrect even though it seems like a reasonable thing to hope for. This is not to say that the author's argument is unreasonable. However, the author is clearly against the proposal, and the question asks us why he is against it, i.e., what he thinks will happen if we try it. What he ultimately wants instead is not part of the question.

(D) is incorrect because it is probably an intended outcome of the proposal. If the author is against the proposal, he would not claim that it will have a desirable outcome.

(E) is incorrect because it states what the author wants. The argument clearly suggests that he thinks the opposite will happen.